What is the next three terms for 7, 21, 63

Answers

Answer 1

Answer:

189, 567, 1701

Step-by-step explanation:

multiply each term by 3.


Related Questions

this is really easy promise

Answers

Answer:

A.) slope= 4 and y intercept= 3

Step-by-step explanation:

y = x4+3

Answer:

y = x4+3

Step-by-step explanation:

look at answer

the temperature at noon was -3°C. By 10 Pm on the same day the temperature decrease by 5.4° what was the temperature at 10 pm​

Answers

Answer:

-8.4

Step-by-step explanation:

Answer:

Probbly -8.4

Step-by-step explanation:

Jill has not been able to maintain the $1,000 minimum balance required to avoid fees on her checking account. She wants to switch to a different account with a fee of $0.20 per check and a $12.50 monthly maintenance fee. Jill wants to estimate the fees for her new account. Below is a summary of the checks she has written from May to August. What is the mean number of checks Jill wrote per month during the last four months? Based on the mean, estimate how much Jill expects to pay in per-check fees each month after she switches to the new account. Estimate the total monthly fees Jill will pay each month for the new checking account.

Answers

Step-by-step explanation:

 Kindly find attached a summary of the checks she has written from May to August.

A. What is the mean number of checks Jill wrote per month during the last four months

mean= (14+19+23+24)/4

mean= 80/4

mean= 20 checks

B.  Based on the mean, estimate how much Jill expects to pay in per-check fees each month after she switches to the new account.

the expression for the total is given as

y=12.50+0.2x

y=12.50+0.2(20)

y=12.50+4

y=16.5

=$16.5

C. Estimate the total monthly fees Jill will pay each month for the new checking account.

May= 12.50+0.2(14)

May=12.5+2.8= 15.3

June= 12.50+0.2(19)

June=12.5+3.8= 16.3

July= 12.50+0.2(23)

July=12.5+4.6= 17.1

August= 12.50+0.2(24)

August=12.5+4.8= 17.3

total will be = 15.3+16.3+17.1+17.3= $66

A:20

B:$4.00

C:$16.50

A: 14+19+23+24/4 = 20

B: 20/.2 = 4

C: 12.50+4= 16.50

Find the value of x.

Answers

Answer:

12

Step-by-step explanation:

1. the rule: 3:6=6:x;

2. if 3/6=6/x, then x=12.

What is the leading coefficient of a third degree polynomial function that has an output of 1,272 when x=2, and has zeros of −6, 7i, and −7i?

Answers

Answer:

The leading coefficient is 3

Step-by-step explanation:

Polynomials

Given the roots of a polynomial x1,x2,x3, it can be expressed as:

[tex]p(x)=a(x-x1)(x-x2)(x-x3)[/tex]

Where a is the leading coefficient.

We are given the roots x1=-6, x2=7i, x3=-7i, thus:

[tex]p(x)=a(x+6)(x-7i)(x+7i)[/tex]

Operating the product of the conjugated imaginary roots:

[tex]p(x)=a(x+6)(x^2+49)[/tex]

Knowing p(2)=1,272 we can find the value of a

[tex]p(2)=a(2+6)(4+49)=1,272[/tex]

Operating:

[tex]a(8)(53)=1,272[/tex]

[tex]424a=1,272[/tex]

Solving:

[tex]a=1,272/424[/tex]

a=3

The leading coefficient is 3

Answer:

3

Step-by-step explanation:


Emilia wrote a total of 12 pages over 4 hours. After spending 6 hours writing this week, how
many pages will Emilia have written in total? Assume the relationship is directly proportional.

Answers

Answer:

30 pages

Step-by-step explanation:

Knowing she wrote 12 pages in 4 hours, we can do 12/4 to get 3 pages per hour. Multiply 3 pages by the 6 hours she spends writing and you get 18 pages in 6 hours. Add 12 pages and 18 pages and you have 30 pages.

Anyone know this because I need help !!!!

Answers

Answer: 90 clockwise rotation

Step-by-step explanation:

you can take any point and replace its (x,y) to (y,-x) otherwise called the 90 clockwise conversion and it will map figure C onto figure C'

Describe the similarities and differences between a bar graph and a histogram.​

Answers

Answer:

Bar graphs are used with nominal or ordinal scores; histograms are used with interval or ratio scores.

Step-by-step explanation:

The number of students in a chess club decreased from 16 to 12. What is the percent decrease ? Round to the nearest percent.

Answers

Answer:

25% decrease

- Trust me it's right

You are buying fruit to make fruit baskets. Apples come in bags of 20. Oranges come in bags of 16, and bananas come in bags of 32. You have one bag of each fruit. Each fruit basket must be identical. What is the greatest number of fruit baskets that you can make using all of the fruit? *

Answers

Answer:

4 fruit baskets.

Step-by-step explanation:

Use GCF to find the answer. 20 divided by 4 is 5, 16 divided by 4 is 4, and 32 divided by 4 is 8.

suppose the retail price of an automobile is $28,000 in 1998 and that it increase at 3% per year. a) write a equation exponential function y=a(1 +r) b) use the model to predict the retail of the automobile in 2016

Answers

Answer:

......................................

Walter is helping to make cookies for a basketball tournament. He's made 15 cookies so far. His coach asked him to make at least 20 cookies but no more than 55. Solve the inequality and interpret the solution.

Answers

Answer:

[tex]5\leq x\leq 40[/tex]

Here, [tex]x[/tex] denotes the additional cookies that need to be made by Walter besides 15 cookies made by him so far

Step-by-step explanation:

Number of cookies made by Walter so far = 15

He is asked by his coach to make at least 20 cookies but no more than 55.

Let [tex]x[/tex] denotes the additional cookies that need to be made by Walter besides 15 cookies made by him so far.

The inequality is [tex]20\leq x+15\leq 55[/tex]

Now solve this inequality.

[tex]20\leq x+15\leq 55\\20-15\leq x\leq 55-15\\5\leq x\leq 40[/tex]

Answer: a

Step-by-step explanation: because its right

Solve the following system of equations for X..
3x – 2y = 8
2x + 2y = 7
Help
A)3
B)-3
C)15
D)1

Answers

Answer:

x  =  3

Step-by-step explanation:

Solve for the first variable in one of the equations, then substitute the result into the other equation.

X would be -3so therefore it’s x-3

A company sells widgets. The amount of profit, y, made by the company, is related to the selling price of each widget, x, by the given equation. Using this equation, find out what price the widgets should be sold for, to the nearest cent, for the company to make the maximum profit.
y=-10x^2+600x-3588
y=−10x
2
+600x−3588

Answers

Answer:

Step-by-step explanation:

The maximum profit will be found in the vertex of the parabola, which is what your equation is. You could do this by completing the square, but it is way easier to just solve for h and k using the following formulas:

[tex]h=\frac{-b}{2a}[/tex] for the x coordinate of the vertex, and

[tex]k=c-\frac{b^2}{4a}[/tex] for the y coordinate of the vertex.

x will be the selling price of each widget and y will be the profit. Usually, x is the number of the items sold, but I'm going off your info here for what the vertex means in the context of this problem.

Our variables for the quadratic are as follows:

a = -10

b = 600

c = -3588. Therefore,

[tex]h=\frac{-600}{2(-10)}=30[/tex] so the cost of each widget is $30. Now for the profit:

[tex]k=-3588-(\frac{(600)^2}{4(-10)})[/tex] This one is worth the simplification step by step:

[tex]k=-3588-(\frac{360000}{-40})[/tex] and

k = -3588 - (-9000) and

k = -3588 + 9000 so

k = 5412

That means that the profit made by selling the widgets at $30 apiece is $5412.

Hence,the profit made by selling the widgets at $[tex]30[/tex] apiece is $[tex]5412[/tex].

What is the maximum profit?

Maximum profit, or profit maximisation, is the process of finding the right price for your products or services to produce the best profit.

Here given that,

A company sells widgets. The amount of profit, [tex]y[/tex], made by the company, is related to the selling price of each widget, [tex]x[/tex], by the given equation.

As the maximum profit found in the vertex of the parabola,

Here, [tex]x[/tex] will be the selling price of each widget and [tex]y[/tex] will be the profit.

The number of items sold is [tex]x[/tex].

So, the quadratic equation is:-

[tex]a = -10b = 600c = -3588.[/tex]

Therefore, so the cost of each widget is $[tex]30[/tex].

For the profit:-

[tex]k = -3588 - (-9000) andk = -3588 + 9000 sok = 5412[/tex]

Hence,the profit made by selling the widgets at $[tex]30[/tex] apiece is $[tex]5412[/tex].

To know more about the maximum profit

https://brainly.com/question/16755335

#SPJ2

In a study using 50 foreign-language vocabulary words, the learning rate L (in words per minute) was found to depend on the number of words already learned, x, according to the equation L(x) = 20 – 0.4x

Answers

Here is the complete question

1. State coordinates of x axis

2. State coordinates of y axis

3. State the slope

4. Graph the function

5. Is learning rate increasing or decreasing?

Answer:

1. (50,0)

2. (0,20)

3. Slope = -0.4

4. Check attachment for graph

5. Learning rate is decreasing

Step-by-step explanation:

1.

L(x) = 20-0.4x

20 = 0.4x

x = 20/0.4

x = 50

(50,0)

2

For y coordinate;

L(0) = 20-0.4(0)

= 20

(0,20)

3.

Slope,

Y = -0.4x+20

We compare with

Y = mx+c

m = -0.4

Intercept c = 20

4. Check attachment

5. The slope is negative at m = -0.4 therefore the rate of learning is on a decline. As number of words learnt increases, learning rate falls.

Answer:

Step-by-step explanation:

What are the true and false statements for If two planes intersect, then their intersection is a line. Include converse, inverse and contrapositive.

Answers

Answer and Step-by-step explanation:

Solution:

Statement:

If two planes intersect  their intersection is a line.

Suppose that

P =if two planes intersect.

q = then their intersection is line.

¬p = if two planes do not intersect.

¬q = then their intersection is not a line.

Converse:

If two planes intersect their intersection is a line.

P → q

Inverse:

If two planes do not intersect, then their intersection is not a line.

¬p → ¬q

Contrapositive:

If two planes intersection is not a line, then they do not intersect.

¬q → ¬p

f(x) = x2. What is g(x)?

Answers

Answer:

B. g(x)=(2x)^2

Step-by-step explanation:

this is right

A sinking fund is established to discharge a debt of $30,000 in 5 years. If deposits are made at the end of each 6-month period and interest is paid at the rate of 4%, compounded semiannually, what is the amount of each deposit?

Answers

Answer:

what you have to do is do 30000 * 5 and then do 6 divided by the number and then four times that number and whatever the answer you get be your answer hope this help

The frequency of the red eyes (p allele) in fruit flies is 63% what is the frequency of the white eye allele (9 allele)?

37%

47%

57%

23%

Answers

Answer:

[tex]q = 37\%[/tex]

Step-by-step explanation:

Given

Red Eyes

[tex]p = 63\%[/tex]

Required

Determine Frequency of White Eyes (q)

In probability;

[tex]p + q = 1[/tex]

Make q the subject

[tex]q = 1 - p[/tex]

Substitute 63% for p

[tex]q = 1 - 63\%[/tex]

[tex]q = 37\%[/tex]

Answer:

37%

Step-by-step explanation:

If 10% of x is 20, what is 23% of x? A. 33 B. 46 C. 200 D. 230

Answers

The correct answer is B

Answer:

B. 46

Step-by-step explanation:

20*10=200

200*0.23=46

Point J is on line segment IK. Given JK = 2x, IJ = 5x, and IK = x + 6, determine the numerical length of JK.

Answers

Answer:

[tex]JK = 2[/tex]

Step-by-step explanation:

Given:

[tex]JK = 2x[/tex]

[tex]IJ = 5x[/tex]

[tex]IK = x + 6[/tex]

Required

Solve for JK.

Since J is on IK, we have:

[tex]IK = IJ + JK[/tex]

[tex]x + 6 = 5x + 2x[/tex]

[tex]x + 6 = 7x[/tex]

Collect Like Terms

[tex]7x - x = 6[/tex]

[tex]6x = 6[/tex]

Solve for x

[tex]x = 6/6[/tex]

[tex]x = 1[/tex]

Substitute 1 for x in [tex]JK = 2x[/tex]

[tex]JK = 2 * 1[/tex]

[tex]JK = 2[/tex]

The width of a rectangle
is 11 feet longer than the
length. The perimeter of the
rectangle is 70 feet. Find
the length and the width

Answers

Answer:

Hello! The answer to this problem is Length= 27 and Width= 8

Step-by-step explanation:

2(11+2x) +2x = 70

6x + 22 = 70

6x = 48

x = 8

How do we know if a function is a parabola like for instance, this one x^2 + x?

Answers

There is an easy way to tell whether the graph of a quadratic function opens upward or downward: if the leading coefficient is greater than zero, the parabola opens upward, and if the leading coefficient is less than zero, the parabola opens downward.

Select the correct answers.

Mila is at a flea market. She has $50 in her wallet. She decides that she will spend $15 on jewelry, $20 on a pair of jeans, $5 on a T-shirt, and $10 on something to eat. She likes a one-of-a-kind T-shirt, but the seller is not ready to sell it for less than $8. She thinks of five ways to deal with this situation. Which two choices indicate a trade-off?

She will try to find another seller selling the T-shirt for a lower price.

She will give up on the T-shirt and use the money to buy better jeans.

She will use some of the money she received from her aunt on her birthday.

She will pay the seller $5 and promise to pay the remainder on another day.

She will buy the T-shirt for $8 and not order juice at lunch.


Answers

Answer:

1and3

Step-by-step explanation:

QUESTION 16

What is the value of m If the ratios 3:18 and m:42 are in

proportion?

Answers

Answer:

[tex]m = 7[/tex]

Step-by-step explanation:

Given

[tex]3:18 = m:42[/tex]

Required

Solve for m

[tex]3:18 = m:42[/tex]

Convert to fraction

[tex]\frac{3}{18} = \frac{m}{42}[/tex]

Multiply both sides by 42

[tex]42 * \frac{3}{18} = \frac{m}{42} * 42[/tex]

[tex]42 * \frac{3}{18} = m[/tex]

[tex]m = 42 * \frac{3}{18}[/tex]

[tex]m = \frac{42 * 3}{18}[/tex]

[tex]m = \frac{126}{18}[/tex]

[tex]m = 7[/tex]

How do you check if a number is rational?
How do you check if a number is irrational?



Thanks!

P.S. 50 points & brainiest to whoever answer first!

Answers

Answer:

well it's really simple

Step-by-step explanation:

So the quick and dirty tip for checking whether a number is rational or irrational is to write it in decimal form. If the decimal goes on and on forever and never stops or begins to repeat predictably, it’s irrational. If the decimal stops after a finite number of digits or begins to repeat predictably, it’s rational.

Any number that is able to be written down as a fraction/ratio is a ration number.

An irational number is a number hat has a decimal, but can't be written as a fraction. Ex: Pi/π

Simplify the expression 4x+3(5y+x

Answers

Answer:

Step-by-step explanation:

4x + 3(5y + x)

= 4x (5y + x) + 3 (5y + x)

= 20xy + 4x² + 15y + 3x

6th grade math I mark as brainliest

Answers

The answer is 9 hope this helps

Which equation represents a line that is perpendicular to the line on the graph ? O y = - 4x + 3 O O y = - 1/4 * x + 1 y = 1/4 * x - 2 O O y = 4x + 4

Answers

Answer:

D

Step-by-step explanation:

Answer

Its  C notD!!!!!!

Step-by-step explanation:

The other person is wrong im right

Why did karen take the kids (;´༎ຶД༎ຶ`)

Answers

haha life question right there

Wait Jimmy what are you doing here I didn't expect this

so we meet again, brother.

Answer:

Because Karen found your briefcase of $1 bills and some crusty socks.

Get over the divorce Jimmy, she doesn't like fax machines.

Other Questions
HELP MEEE !!!How do scientists study the earth's core?O Using SONARO Using SubmarinesO Using Seismic Wave dataO The earth's core cannot be studied why is it quieter in ereas wiyh high evevation such as tagaytay and baguio than in lowlandspaki answer pls. Which type of force enables all the matter made up of atoms to exist?A. ElectromagneticB. NormalC. Strong nuclearD. Gravitational what is the slope of (19,14) & (21,19) Suppose that a late succession plant that is surrounded by early succession plants grows at a rate of 0.5 cm per day. If the early succession plants are removed experimentally, the late succession plants are observed to grow at a rate of 0.9 cm per day. This experiment shows that the early succession plants have a _______ effect on the late succession plants, and this observation is most consistent with the _______ model of succession. john goes out to eat at a local restaurant the bill comes to $47.38 john wants to leave a 20% tip how much of a tip should he leave? A football team loses 5 yards on one play and then loses 8 yards on the next play.a. How many yards did they lose in two plays?b. How do we represent a loss of 5 yards?c. How do we represent a loss of 8 yards?d. What does our addition sentence look like?e. What is the answer to the question? How did Spain support the colonists at the beginning of the American Revolution? according to the law of demand consumer demand for a product will decrease if Nancy has 192 golf balls.How many dozen golf balls does she have? 3x34343334445877958689 Charlene has a BMI of 29; therefore, she wants to change her body composition. Which plan would be BEST for her? A. She should exercise 60 minutes a day, eat six small meals that equal less than 2,000 calories, and make sure to get plenty of rest. B. She should run 30 minutes a day, eat balanced meals that contain about 1,000 calories each, and sleep about six hours a night. C. She should lift weights 45 minutes a day, eat 4,000 calories a day, and sleep at least 10 hours a night. D. She should do cardiovascular exercises, eat six small meals that altogether equal 1,000 calories, and sleep about eight hours a night. Question 2 1 ptsComplete the following sentences with one of the given options:A Ernesto y Emilia les gusta ir al ro, as que _____.Group of answer choicestienen reservaciones en el hotel.van a pescarnecesitan tarjeta de crditoFlag this QuestionQuestion 3 1 ptsA Emilia y Ernesto les encanta el arte, as que _____.Group of answer choicesvan a acamparhacen una excursinvisitan un museoFlag this QuestionQuestion 4 1 ptsEmilia quiere comprar un anillo, aretes y un collar en el mercado al aire libre; le gustan _____.Group of answer choiceslas tarjetas de crditolos museoslas joyasFlag this QuestionQuestion 5 1 ptsErnesto est de vacaciones en otro pas; es un _____.Group of answer choicesturistacaballodinero en efectivoFlag this QuestionQuestion 6 1 ptsLa familia Salinas necesita _____________ para descansar.Group of answer choicesir a pescarun alojamientoir al museoFlag this QuestionQuestion 7 1 ptsAna y Jorge reciben las llaves de las habitaciones en la _____________.Group of answer choicesel mercado de artesanasacamparrecepcin del hotelFlag this QuestionQuestion 8 1 ptsFederico les quiere escribir a sus amigos; les va a mandar __________.Group of answer choicesuna tarjeta postalva tomar fotoshacer una reservacinFlag this QuestionQuestion 9 1 ptsQuiero comprar ____ para mi mam. Necesito ir al mercado de artesanas.Group of answer choicesregatearrecuerdostarjetas de crditoFlag this QuestionQuestion 10 1 ptsAceptan _________? Yo NO tengo dinero en efectivo.Group of answer choicesmuseosartesanastarjetas de crditoFlag this QuestionQuestion 11 1 ptsPagaste demasiado por los aretes. T NO sabes __________ .Group of answer choicesregatearpescardinero en efectivoFlag this QuestionQuestion 12 1 ptsCuando entro en el hotel, tomo ____ para ir a mi habitacin.Group of answer choicesel autobsel ascensorel hostal POSSIBLE POINTS: 0.86 Marcus is trying to decide between a cable subscription and a satellite dish. The cable subscription has a $40 setup fee and costs an additional $38 per month. Marcus has $500 budgeted for cable. How many months (m) can he pay for the cable subscription? WRITE AN EQUATION THAT CAN BE USED TO FIND THE NUMBER OF MONTHS HE CAN PAY FOR THE CABLE SUBSCRIPTION Please help me with this. First answer gets brainliest what is this a picture of? Jamal needs to dig a ditch 15 yards long.Jamal digs 7 yards in the morning andanother 3 yards after lunch. How much is left to digmore of the ditch is left to dig? Marti wants to buy a dress priced at $89.75. If the sales tax is 8%, what is the total amount she must pay for the dress? A car dealer acquires a used car for $12,000, with terms FOB shipping point. Compute total inventory costs assigned to the used car if additional costs include: $100 for transportation-in. $170 for shipping insurance. $800 for car import duties. $140 for advertising. $1,400 for sales staff salaries. $150 for trimming shrubs.Required:For computing inventory, what cost is assigned to the used car? 50 POINTS please show the step by step process.I already know the answers i just the steps!!1.Factor: 4x^2+28x-362.Factor: 125x^6-27y^153.Multiply: (4x-7)(5x-8)